ChaseDream

标题: OG12-48 [打印本页]

作者: zubin1988    时间: 2010-11-10 22:01
标题: OG12-48
题目:A  researcher discovered that people who have low levels of immune-system activity tend to score much lower on tests of mental health than do people with normal or high immune-system activity. The researcher concluded from this experiment that the immune system protects against mental illness as well as against physical disease

The researcher's conclusion depends on which of the following assumption?
A.High immune-system activity protects against mental illness better than normal immune-system activity does.
B.mental illness is similar to physical disease in its effects on body systems.
C.people with high immune-system activity cannot develop mental illness.
D.mental illness does not cause people's immune-system activity to decrease.
E.psychological treatment of mental illness is not as effective as is medical treatmen.


这题的答案选C,是怎么的出来的呢?
作者: sdcar2010    时间: 2010-11-10 22:19
D is the right answer.
作者: sdcar2010    时间: 2010-11-10 22:20
This is a typical correlation and causation question.  If two events A and B happen at the same time, there are several possibilities:  A causes B; B causes A; another event C causes A or B or both!




欢迎光临 ChaseDream (https://forum.chasedream.com/) Powered by Discuz! X3.3